Verify that $H(x-ct)$ is a weak solution of wave equation $u_tt=c^2u_xx$weak solution of wave equationTwo weak formulations for a time-dependent PDE: are they equivalent?Fundamental solution of wave equation in 3Dreversibility scalar conservation lawweak solution of wave equationIntegration by parts in time derivative of energy integral for wave equationFinite speed of propagation for weak solutions?Entropy solution to scalar conservation lawConvergence of integral (Lebesgue and Bochner spaces)Evans PDE Conservation Law IntegralEnergy equalities and estimates for weak solutions

How can we generalize the fact of finite dimensional vector space to an infinte dimensional case?

Why did the HMS Bounty go back to a time when whales are already rare?

Yosemite Fire Rings - What to Expect?

How could a planet have erratic days?

Is there a single word describing earning money through any means?

Longest common substring in linear time

Where did Heinlein say "Once you get to Earth orbit, you're halfway to anywhere in the Solar System"?

The IT department bottlenecks progress. How should I handle this?

When were female captains banned from Starfleet?

Which one is correct as adjective “protruding” or “protruded”?

Why is it that I can sometimes guess the next note?

What is the evidence for the "tyranny of the majority problem" in a direct democracy context?

Why should universal income be universal?

Electoral considerations aside, what are potential benefits, for the US, of policy changes proposed by the tweet recognizing Golan annexation?

Argument list too long when zipping large list of certain files in a folder

Non-trope happy ending?

Store Credit Card Information in Password Manager?

Open a doc from terminal, but not by its name

On a tidally locked planet, would time be quantized?

Creepy dinosaur pc game identification

Melting point of aspirin, contradicting sources

Why Shazam when there is already Superman?

How can Trident be so inexpensive? Will it orbit Triton or just do a (slow) flyby?

Why does the Sun have different day lengths, but not the gas giants?



Verify that $H(x-ct)$ is a weak solution of wave equation $u_tt=c^2u_xx$


weak solution of wave equationTwo weak formulations for a time-dependent PDE: are they equivalent?Fundamental solution of wave equation in 3Dreversibility scalar conservation lawweak solution of wave equationIntegration by parts in time derivative of energy integral for wave equationFinite speed of propagation for weak solutions?Entropy solution to scalar conservation lawConvergence of integral (Lebesgue and Bochner spaces)Evans PDE Conservation Law IntegralEnergy equalities and estimates for weak solutions













3












$begingroup$


The problem is in Strauss, Partial Diferential Equations 2nd edition, 12.1 Ex.5.



I want to verify by direct substitution that Heaviside DISTRIBUTION $H(x-ct)$ is a weak solution of wave equation $u_tt=c^2u_xx$.



I found almost identical question at weak solution of wave equation here but I think my problem is slightly different from this, since I considered that $(x,t)inmathbb Rtimes(mathbb R^+cup0)$.



$$int_infty^infty int_0^infty H(x-ct)(phi_tt-c^2phi_xx)dtdx=0$$



should be true for all $phiinmathcal D(mathbb Rtimes(mathbb R^+))$.



Using $phi$ is a $mathcal C^infty$ function with compact support, I found it can be reduced to



$$
beginalign
& int_0^inftyint_0^x/c phi_ttdtdx - c^2int_0^inftyint_ct^infty phi_xxdxdt \
= & int_0^infty (phi_t (x, dfracxc) - phi_t (x, 0))dx + cint_0^inftyphi_x(t,dfractc)dt \
= & int_0^infty cdfracdphids(s,dfracsc)ds-int_0^infty phi_t (x, 0)dx \
= & -cphi(0,0) - int_0^infty phi_t (x, 0)dx.
endalign$$



Since the second term cannot be integrated explicitly, I considered a closed curve on $mathbb Rtimes(mathbb R^+cup0)$ which connects $(0,0),(X,0),(X,T),(0,T)$ by line segments, then send $X$ and $T$ to $+infty$. Since $phi$ and its derivatives vanish over some radius R, the second term would be $-phi(0,0)$, not $-cphi(0,0)$.



Maybe I has made a mistake on integration, but can't find that. Could anyone give me some help?










share|cite|improve this question











$endgroup$











  • $begingroup$
    If $phi in mathcal D(mathbb R_x times mathbb R^+_t)$, does this mean that the support of $phi$ in time is bounded away from 0?
    $endgroup$
    – Calvin Khor
    Mar 15 at 18:56










  • $begingroup$
    @CalvinKhor I meant that $phi$ is a test function in distribution space en.wikipedia.org/wiki/Distribution_(mathematics). So $phi$ and its derivatives will be identically zero outside some bounded interval.
    $endgroup$
    – Jinmu You
    Mar 16 at 2:23






  • 1




    $begingroup$
    I know what a distribution and what a test function is, I'm asking you to check that you mean $t=0$ is included in the support of $phi$. In this case, i think the definition of a weak solution should also have some terms with $t=0$ from the integration by parts
    $endgroup$
    – Calvin Khor
    Mar 16 at 8:31











  • $begingroup$
    @CalvinKhor Sorry for misunderstanding. I think I cannot be sure that $t=0$ is included in the support of $phi$. Maybe $phi(0,0)$ can get some nonzero value. My expectation was that the last integral term would be $-cphi(0,0)$ and make the integration zero, regardless of the $phi$. But I can't justify that.
    $endgroup$
    – Jinmu You
    Mar 16 at 14:26
















3












$begingroup$


The problem is in Strauss, Partial Diferential Equations 2nd edition, 12.1 Ex.5.



I want to verify by direct substitution that Heaviside DISTRIBUTION $H(x-ct)$ is a weak solution of wave equation $u_tt=c^2u_xx$.



I found almost identical question at weak solution of wave equation here but I think my problem is slightly different from this, since I considered that $(x,t)inmathbb Rtimes(mathbb R^+cup0)$.



$$int_infty^infty int_0^infty H(x-ct)(phi_tt-c^2phi_xx)dtdx=0$$



should be true for all $phiinmathcal D(mathbb Rtimes(mathbb R^+))$.



Using $phi$ is a $mathcal C^infty$ function with compact support, I found it can be reduced to



$$
beginalign
& int_0^inftyint_0^x/c phi_ttdtdx - c^2int_0^inftyint_ct^infty phi_xxdxdt \
= & int_0^infty (phi_t (x, dfracxc) - phi_t (x, 0))dx + cint_0^inftyphi_x(t,dfractc)dt \
= & int_0^infty cdfracdphids(s,dfracsc)ds-int_0^infty phi_t (x, 0)dx \
= & -cphi(0,0) - int_0^infty phi_t (x, 0)dx.
endalign$$



Since the second term cannot be integrated explicitly, I considered a closed curve on $mathbb Rtimes(mathbb R^+cup0)$ which connects $(0,0),(X,0),(X,T),(0,T)$ by line segments, then send $X$ and $T$ to $+infty$. Since $phi$ and its derivatives vanish over some radius R, the second term would be $-phi(0,0)$, not $-cphi(0,0)$.



Maybe I has made a mistake on integration, but can't find that. Could anyone give me some help?










share|cite|improve this question











$endgroup$











  • $begingroup$
    If $phi in mathcal D(mathbb R_x times mathbb R^+_t)$, does this mean that the support of $phi$ in time is bounded away from 0?
    $endgroup$
    – Calvin Khor
    Mar 15 at 18:56










  • $begingroup$
    @CalvinKhor I meant that $phi$ is a test function in distribution space en.wikipedia.org/wiki/Distribution_(mathematics). So $phi$ and its derivatives will be identically zero outside some bounded interval.
    $endgroup$
    – Jinmu You
    Mar 16 at 2:23






  • 1




    $begingroup$
    I know what a distribution and what a test function is, I'm asking you to check that you mean $t=0$ is included in the support of $phi$. In this case, i think the definition of a weak solution should also have some terms with $t=0$ from the integration by parts
    $endgroup$
    – Calvin Khor
    Mar 16 at 8:31











  • $begingroup$
    @CalvinKhor Sorry for misunderstanding. I think I cannot be sure that $t=0$ is included in the support of $phi$. Maybe $phi(0,0)$ can get some nonzero value. My expectation was that the last integral term would be $-cphi(0,0)$ and make the integration zero, regardless of the $phi$. But I can't justify that.
    $endgroup$
    – Jinmu You
    Mar 16 at 14:26














3












3








3


2



$begingroup$


The problem is in Strauss, Partial Diferential Equations 2nd edition, 12.1 Ex.5.



I want to verify by direct substitution that Heaviside DISTRIBUTION $H(x-ct)$ is a weak solution of wave equation $u_tt=c^2u_xx$.



I found almost identical question at weak solution of wave equation here but I think my problem is slightly different from this, since I considered that $(x,t)inmathbb Rtimes(mathbb R^+cup0)$.



$$int_infty^infty int_0^infty H(x-ct)(phi_tt-c^2phi_xx)dtdx=0$$



should be true for all $phiinmathcal D(mathbb Rtimes(mathbb R^+))$.



Using $phi$ is a $mathcal C^infty$ function with compact support, I found it can be reduced to



$$
beginalign
& int_0^inftyint_0^x/c phi_ttdtdx - c^2int_0^inftyint_ct^infty phi_xxdxdt \
= & int_0^infty (phi_t (x, dfracxc) - phi_t (x, 0))dx + cint_0^inftyphi_x(t,dfractc)dt \
= & int_0^infty cdfracdphids(s,dfracsc)ds-int_0^infty phi_t (x, 0)dx \
= & -cphi(0,0) - int_0^infty phi_t (x, 0)dx.
endalign$$



Since the second term cannot be integrated explicitly, I considered a closed curve on $mathbb Rtimes(mathbb R^+cup0)$ which connects $(0,0),(X,0),(X,T),(0,T)$ by line segments, then send $X$ and $T$ to $+infty$. Since $phi$ and its derivatives vanish over some radius R, the second term would be $-phi(0,0)$, not $-cphi(0,0)$.



Maybe I has made a mistake on integration, but can't find that. Could anyone give me some help?










share|cite|improve this question











$endgroup$




The problem is in Strauss, Partial Diferential Equations 2nd edition, 12.1 Ex.5.



I want to verify by direct substitution that Heaviside DISTRIBUTION $H(x-ct)$ is a weak solution of wave equation $u_tt=c^2u_xx$.



I found almost identical question at weak solution of wave equation here but I think my problem is slightly different from this, since I considered that $(x,t)inmathbb Rtimes(mathbb R^+cup0)$.



$$int_infty^infty int_0^infty H(x-ct)(phi_tt-c^2phi_xx)dtdx=0$$



should be true for all $phiinmathcal D(mathbb Rtimes(mathbb R^+))$.



Using $phi$ is a $mathcal C^infty$ function with compact support, I found it can be reduced to



$$
beginalign
& int_0^inftyint_0^x/c phi_ttdtdx - c^2int_0^inftyint_ct^infty phi_xxdxdt \
= & int_0^infty (phi_t (x, dfracxc) - phi_t (x, 0))dx + cint_0^inftyphi_x(t,dfractc)dt \
= & int_0^infty cdfracdphids(s,dfracsc)ds-int_0^infty phi_t (x, 0)dx \
= & -cphi(0,0) - int_0^infty phi_t (x, 0)dx.
endalign$$



Since the second term cannot be integrated explicitly, I considered a closed curve on $mathbb Rtimes(mathbb R^+cup0)$ which connects $(0,0),(X,0),(X,T),(0,T)$ by line segments, then send $X$ and $T$ to $+infty$. Since $phi$ and its derivatives vanish over some radius R, the second term would be $-phi(0,0)$, not $-cphi(0,0)$.



Maybe I has made a mistake on integration, but can't find that. Could anyone give me some help?







pde






share|cite|improve this question















share|cite|improve this question













share|cite|improve this question




share|cite|improve this question








edited Mar 15 at 18:45









Spencer

8,65012056




8,65012056










asked Mar 15 at 18:13









Jinmu YouJinmu You

339210




339210











  • $begingroup$
    If $phi in mathcal D(mathbb R_x times mathbb R^+_t)$, does this mean that the support of $phi$ in time is bounded away from 0?
    $endgroup$
    – Calvin Khor
    Mar 15 at 18:56










  • $begingroup$
    @CalvinKhor I meant that $phi$ is a test function in distribution space en.wikipedia.org/wiki/Distribution_(mathematics). So $phi$ and its derivatives will be identically zero outside some bounded interval.
    $endgroup$
    – Jinmu You
    Mar 16 at 2:23






  • 1




    $begingroup$
    I know what a distribution and what a test function is, I'm asking you to check that you mean $t=0$ is included in the support of $phi$. In this case, i think the definition of a weak solution should also have some terms with $t=0$ from the integration by parts
    $endgroup$
    – Calvin Khor
    Mar 16 at 8:31











  • $begingroup$
    @CalvinKhor Sorry for misunderstanding. I think I cannot be sure that $t=0$ is included in the support of $phi$. Maybe $phi(0,0)$ can get some nonzero value. My expectation was that the last integral term would be $-cphi(0,0)$ and make the integration zero, regardless of the $phi$. But I can't justify that.
    $endgroup$
    – Jinmu You
    Mar 16 at 14:26

















  • $begingroup$
    If $phi in mathcal D(mathbb R_x times mathbb R^+_t)$, does this mean that the support of $phi$ in time is bounded away from 0?
    $endgroup$
    – Calvin Khor
    Mar 15 at 18:56










  • $begingroup$
    @CalvinKhor I meant that $phi$ is a test function in distribution space en.wikipedia.org/wiki/Distribution_(mathematics). So $phi$ and its derivatives will be identically zero outside some bounded interval.
    $endgroup$
    – Jinmu You
    Mar 16 at 2:23






  • 1




    $begingroup$
    I know what a distribution and what a test function is, I'm asking you to check that you mean $t=0$ is included in the support of $phi$. In this case, i think the definition of a weak solution should also have some terms with $t=0$ from the integration by parts
    $endgroup$
    – Calvin Khor
    Mar 16 at 8:31











  • $begingroup$
    @CalvinKhor Sorry for misunderstanding. I think I cannot be sure that $t=0$ is included in the support of $phi$. Maybe $phi(0,0)$ can get some nonzero value. My expectation was that the last integral term would be $-cphi(0,0)$ and make the integration zero, regardless of the $phi$. But I can't justify that.
    $endgroup$
    – Jinmu You
    Mar 16 at 14:26
















$begingroup$
If $phi in mathcal D(mathbb R_x times mathbb R^+_t)$, does this mean that the support of $phi$ in time is bounded away from 0?
$endgroup$
– Calvin Khor
Mar 15 at 18:56




$begingroup$
If $phi in mathcal D(mathbb R_x times mathbb R^+_t)$, does this mean that the support of $phi$ in time is bounded away from 0?
$endgroup$
– Calvin Khor
Mar 15 at 18:56












$begingroup$
@CalvinKhor I meant that $phi$ is a test function in distribution space en.wikipedia.org/wiki/Distribution_(mathematics). So $phi$ and its derivatives will be identically zero outside some bounded interval.
$endgroup$
– Jinmu You
Mar 16 at 2:23




$begingroup$
@CalvinKhor I meant that $phi$ is a test function in distribution space en.wikipedia.org/wiki/Distribution_(mathematics). So $phi$ and its derivatives will be identically zero outside some bounded interval.
$endgroup$
– Jinmu You
Mar 16 at 2:23




1




1




$begingroup$
I know what a distribution and what a test function is, I'm asking you to check that you mean $t=0$ is included in the support of $phi$. In this case, i think the definition of a weak solution should also have some terms with $t=0$ from the integration by parts
$endgroup$
– Calvin Khor
Mar 16 at 8:31





$begingroup$
I know what a distribution and what a test function is, I'm asking you to check that you mean $t=0$ is included in the support of $phi$. In this case, i think the definition of a weak solution should also have some terms with $t=0$ from the integration by parts
$endgroup$
– Calvin Khor
Mar 16 at 8:31













$begingroup$
@CalvinKhor Sorry for misunderstanding. I think I cannot be sure that $t=0$ is included in the support of $phi$. Maybe $phi(0,0)$ can get some nonzero value. My expectation was that the last integral term would be $-cphi(0,0)$ and make the integration zero, regardless of the $phi$. But I can't justify that.
$endgroup$
– Jinmu You
Mar 16 at 14:26





$begingroup$
@CalvinKhor Sorry for misunderstanding. I think I cannot be sure that $t=0$ is included in the support of $phi$. Maybe $phi(0,0)$ can get some nonzero value. My expectation was that the last integral term would be $-cphi(0,0)$ and make the integration zero, regardless of the $phi$. But I can't justify that.
$endgroup$
– Jinmu You
Mar 16 at 14:26











1 Answer
1






active

oldest

votes


















1












$begingroup$

If you want your test functions to have support containing $t=0$, your definition of a weak solution should be adjusted since if $uin C^2$,



$$ int_mathbb Rint_0^infty u_tt(tau) phi(tau) dtau dx = int_mathbb R left( -u_t(0)phi(0) +u(0)phi_t(0) +int_0^infty u(tau)phi_tt(tau) dtau right)dx$$
So for test functions $mathfrak D:=mathcal D(mathbb R times [0,infty))$, we should say that $uinmathfrak D'$ is a weak solution of the IVP for $u_0,u_1inmathcal D(mathbb R)'$,
$$ u_tt = c^2 u_xx\ u(0)=u_0\
u_t(0)=u_1$$

if
$$ -int_mathbb R u_1(x)phi(x,0)dx + int_mathbb R u_0(x)phi_t(x,0)dx +int_mathbb R int_0^infty u(x,t) (phi_tt(x,t)-c^2phi_xx(x,t)) dxdt = 0$$
When $u(x,t)=H(x-ct)$, then $u(x,0)= H(x), u_t(x,0) = -cdelta_0(x),$ so we should use
$$u_0(x) = H(x),\ u_1(x) = -cdelta_0(x). $$
This causes the two extra terms you found to exactly cancel.



For the definition of a weak solution you suggested, you should use the fact that functions in $mathcal D(mathbb R times (0,infty))$ vanish at $t=0$.






share|cite|improve this answer









$endgroup$












  • $begingroup$
    Thanks for perfect explanations. Aside from this problem, does the conclusion $ int_0^infty phi_t(x,0)dx=-phi(0,0) $ when $phiinmathcal D$ makes sense? I got the idea from contour integral in complex analysis, but I feel unsure of using this.
    $endgroup$
    – Jinmu You
    Mar 17 at 12:53







  • 1




    $begingroup$
    @JinmuYou if I understand correctly, the Cauchy's theorem that is required to say integrals on loops are 0 is unavailable in this setting.
    $endgroup$
    – Calvin Khor
    Mar 17 at 13:16










Your Answer





StackExchange.ifUsing("editor", function ()
return StackExchange.using("mathjaxEditing", function ()
StackExchange.MarkdownEditor.creationCallbacks.add(function (editor, postfix)
StackExchange.mathjaxEditing.prepareWmdForMathJax(editor, postfix, [["$", "$"], ["\\(","\\)"]]);
);
);
, "mathjax-editing");

StackExchange.ready(function()
var channelOptions =
tags: "".split(" "),
id: "69"
;
initTagRenderer("".split(" "), "".split(" "), channelOptions);

StackExchange.using("externalEditor", function()
// Have to fire editor after snippets, if snippets enabled
if (StackExchange.settings.snippets.snippetsEnabled)
StackExchange.using("snippets", function()
createEditor();
);

else
createEditor();

);

function createEditor()
StackExchange.prepareEditor(
heartbeatType: 'answer',
autoActivateHeartbeat: false,
convertImagesToLinks: true,
noModals: true,
showLowRepImageUploadWarning: true,
reputationToPostImages: 10,
bindNavPrevention: true,
postfix: "",
imageUploader:
brandingHtml: "Powered by u003ca class="icon-imgur-white" href="https://imgur.com/"u003eu003c/au003e",
contentPolicyHtml: "User contributions licensed under u003ca href="https://creativecommons.org/licenses/by-sa/3.0/"u003ecc by-sa 3.0 with attribution requiredu003c/au003e u003ca href="https://stackoverflow.com/legal/content-policy"u003e(content policy)u003c/au003e",
allowUrls: true
,
noCode: true, onDemand: true,
discardSelector: ".discard-answer"
,immediatelyShowMarkdownHelp:true
);



);













draft saved

draft discarded


















StackExchange.ready(
function ()
StackExchange.openid.initPostLogin('.new-post-login', 'https%3a%2f%2fmath.stackexchange.com%2fquestions%2f3149626%2fverify-that-hx-ct-is-a-weak-solution-of-wave-equation-u-tt-c2u-xx%23new-answer', 'question_page');

);

Post as a guest















Required, but never shown

























1 Answer
1






active

oldest

votes








1 Answer
1






active

oldest

votes









active

oldest

votes






active

oldest

votes









1












$begingroup$

If you want your test functions to have support containing $t=0$, your definition of a weak solution should be adjusted since if $uin C^2$,



$$ int_mathbb Rint_0^infty u_tt(tau) phi(tau) dtau dx = int_mathbb R left( -u_t(0)phi(0) +u(0)phi_t(0) +int_0^infty u(tau)phi_tt(tau) dtau right)dx$$
So for test functions $mathfrak D:=mathcal D(mathbb R times [0,infty))$, we should say that $uinmathfrak D'$ is a weak solution of the IVP for $u_0,u_1inmathcal D(mathbb R)'$,
$$ u_tt = c^2 u_xx\ u(0)=u_0\
u_t(0)=u_1$$

if
$$ -int_mathbb R u_1(x)phi(x,0)dx + int_mathbb R u_0(x)phi_t(x,0)dx +int_mathbb R int_0^infty u(x,t) (phi_tt(x,t)-c^2phi_xx(x,t)) dxdt = 0$$
When $u(x,t)=H(x-ct)$, then $u(x,0)= H(x), u_t(x,0) = -cdelta_0(x),$ so we should use
$$u_0(x) = H(x),\ u_1(x) = -cdelta_0(x). $$
This causes the two extra terms you found to exactly cancel.



For the definition of a weak solution you suggested, you should use the fact that functions in $mathcal D(mathbb R times (0,infty))$ vanish at $t=0$.






share|cite|improve this answer









$endgroup$












  • $begingroup$
    Thanks for perfect explanations. Aside from this problem, does the conclusion $ int_0^infty phi_t(x,0)dx=-phi(0,0) $ when $phiinmathcal D$ makes sense? I got the idea from contour integral in complex analysis, but I feel unsure of using this.
    $endgroup$
    – Jinmu You
    Mar 17 at 12:53







  • 1




    $begingroup$
    @JinmuYou if I understand correctly, the Cauchy's theorem that is required to say integrals on loops are 0 is unavailable in this setting.
    $endgroup$
    – Calvin Khor
    Mar 17 at 13:16















1












$begingroup$

If you want your test functions to have support containing $t=0$, your definition of a weak solution should be adjusted since if $uin C^2$,



$$ int_mathbb Rint_0^infty u_tt(tau) phi(tau) dtau dx = int_mathbb R left( -u_t(0)phi(0) +u(0)phi_t(0) +int_0^infty u(tau)phi_tt(tau) dtau right)dx$$
So for test functions $mathfrak D:=mathcal D(mathbb R times [0,infty))$, we should say that $uinmathfrak D'$ is a weak solution of the IVP for $u_0,u_1inmathcal D(mathbb R)'$,
$$ u_tt = c^2 u_xx\ u(0)=u_0\
u_t(0)=u_1$$

if
$$ -int_mathbb R u_1(x)phi(x,0)dx + int_mathbb R u_0(x)phi_t(x,0)dx +int_mathbb R int_0^infty u(x,t) (phi_tt(x,t)-c^2phi_xx(x,t)) dxdt = 0$$
When $u(x,t)=H(x-ct)$, then $u(x,0)= H(x), u_t(x,0) = -cdelta_0(x),$ so we should use
$$u_0(x) = H(x),\ u_1(x) = -cdelta_0(x). $$
This causes the two extra terms you found to exactly cancel.



For the definition of a weak solution you suggested, you should use the fact that functions in $mathcal D(mathbb R times (0,infty))$ vanish at $t=0$.






share|cite|improve this answer









$endgroup$












  • $begingroup$
    Thanks for perfect explanations. Aside from this problem, does the conclusion $ int_0^infty phi_t(x,0)dx=-phi(0,0) $ when $phiinmathcal D$ makes sense? I got the idea from contour integral in complex analysis, but I feel unsure of using this.
    $endgroup$
    – Jinmu You
    Mar 17 at 12:53







  • 1




    $begingroup$
    @JinmuYou if I understand correctly, the Cauchy's theorem that is required to say integrals on loops are 0 is unavailable in this setting.
    $endgroup$
    – Calvin Khor
    Mar 17 at 13:16













1












1








1





$begingroup$

If you want your test functions to have support containing $t=0$, your definition of a weak solution should be adjusted since if $uin C^2$,



$$ int_mathbb Rint_0^infty u_tt(tau) phi(tau) dtau dx = int_mathbb R left( -u_t(0)phi(0) +u(0)phi_t(0) +int_0^infty u(tau)phi_tt(tau) dtau right)dx$$
So for test functions $mathfrak D:=mathcal D(mathbb R times [0,infty))$, we should say that $uinmathfrak D'$ is a weak solution of the IVP for $u_0,u_1inmathcal D(mathbb R)'$,
$$ u_tt = c^2 u_xx\ u(0)=u_0\
u_t(0)=u_1$$

if
$$ -int_mathbb R u_1(x)phi(x,0)dx + int_mathbb R u_0(x)phi_t(x,0)dx +int_mathbb R int_0^infty u(x,t) (phi_tt(x,t)-c^2phi_xx(x,t)) dxdt = 0$$
When $u(x,t)=H(x-ct)$, then $u(x,0)= H(x), u_t(x,0) = -cdelta_0(x),$ so we should use
$$u_0(x) = H(x),\ u_1(x) = -cdelta_0(x). $$
This causes the two extra terms you found to exactly cancel.



For the definition of a weak solution you suggested, you should use the fact that functions in $mathcal D(mathbb R times (0,infty))$ vanish at $t=0$.






share|cite|improve this answer









$endgroup$



If you want your test functions to have support containing $t=0$, your definition of a weak solution should be adjusted since if $uin C^2$,



$$ int_mathbb Rint_0^infty u_tt(tau) phi(tau) dtau dx = int_mathbb R left( -u_t(0)phi(0) +u(0)phi_t(0) +int_0^infty u(tau)phi_tt(tau) dtau right)dx$$
So for test functions $mathfrak D:=mathcal D(mathbb R times [0,infty))$, we should say that $uinmathfrak D'$ is a weak solution of the IVP for $u_0,u_1inmathcal D(mathbb R)'$,
$$ u_tt = c^2 u_xx\ u(0)=u_0\
u_t(0)=u_1$$

if
$$ -int_mathbb R u_1(x)phi(x,0)dx + int_mathbb R u_0(x)phi_t(x,0)dx +int_mathbb R int_0^infty u(x,t) (phi_tt(x,t)-c^2phi_xx(x,t)) dxdt = 0$$
When $u(x,t)=H(x-ct)$, then $u(x,0)= H(x), u_t(x,0) = -cdelta_0(x),$ so we should use
$$u_0(x) = H(x),\ u_1(x) = -cdelta_0(x). $$
This causes the two extra terms you found to exactly cancel.



For the definition of a weak solution you suggested, you should use the fact that functions in $mathcal D(mathbb R times (0,infty))$ vanish at $t=0$.







share|cite|improve this answer












share|cite|improve this answer



share|cite|improve this answer










answered Mar 16 at 20:29









Calvin KhorCalvin Khor

12.4k21439




12.4k21439











  • $begingroup$
    Thanks for perfect explanations. Aside from this problem, does the conclusion $ int_0^infty phi_t(x,0)dx=-phi(0,0) $ when $phiinmathcal D$ makes sense? I got the idea from contour integral in complex analysis, but I feel unsure of using this.
    $endgroup$
    – Jinmu You
    Mar 17 at 12:53







  • 1




    $begingroup$
    @JinmuYou if I understand correctly, the Cauchy's theorem that is required to say integrals on loops are 0 is unavailable in this setting.
    $endgroup$
    – Calvin Khor
    Mar 17 at 13:16
















  • $begingroup$
    Thanks for perfect explanations. Aside from this problem, does the conclusion $ int_0^infty phi_t(x,0)dx=-phi(0,0) $ when $phiinmathcal D$ makes sense? I got the idea from contour integral in complex analysis, but I feel unsure of using this.
    $endgroup$
    – Jinmu You
    Mar 17 at 12:53







  • 1




    $begingroup$
    @JinmuYou if I understand correctly, the Cauchy's theorem that is required to say integrals on loops are 0 is unavailable in this setting.
    $endgroup$
    – Calvin Khor
    Mar 17 at 13:16















$begingroup$
Thanks for perfect explanations. Aside from this problem, does the conclusion $ int_0^infty phi_t(x,0)dx=-phi(0,0) $ when $phiinmathcal D$ makes sense? I got the idea from contour integral in complex analysis, but I feel unsure of using this.
$endgroup$
– Jinmu You
Mar 17 at 12:53





$begingroup$
Thanks for perfect explanations. Aside from this problem, does the conclusion $ int_0^infty phi_t(x,0)dx=-phi(0,0) $ when $phiinmathcal D$ makes sense? I got the idea from contour integral in complex analysis, but I feel unsure of using this.
$endgroup$
– Jinmu You
Mar 17 at 12:53





1




1




$begingroup$
@JinmuYou if I understand correctly, the Cauchy's theorem that is required to say integrals on loops are 0 is unavailable in this setting.
$endgroup$
– Calvin Khor
Mar 17 at 13:16




$begingroup$
@JinmuYou if I understand correctly, the Cauchy's theorem that is required to say integrals on loops are 0 is unavailable in this setting.
$endgroup$
– Calvin Khor
Mar 17 at 13:16

















draft saved

draft discarded
















































Thanks for contributing an answer to Mathematics Stack Exchange!


  • Please be sure to answer the question. Provide details and share your research!

But avoid


  • Asking for help, clarification, or responding to other answers.

  • Making statements based on opinion; back them up with references or personal experience.

Use MathJax to format equations. MathJax reference.


To learn more, see our tips on writing great answers.




draft saved


draft discarded














StackExchange.ready(
function ()
StackExchange.openid.initPostLogin('.new-post-login', 'https%3a%2f%2fmath.stackexchange.com%2fquestions%2f3149626%2fverify-that-hx-ct-is-a-weak-solution-of-wave-equation-u-tt-c2u-xx%23new-answer', 'question_page');

);

Post as a guest















Required, but never shown





















































Required, but never shown














Required, but never shown












Required, but never shown







Required, but never shown

































Required, but never shown














Required, but never shown












Required, but never shown







Required, but never shown







Popular posts from this blog

Lowndes Grove History Architecture References Navigation menu32°48′6″N 79°57′58″W / 32.80167°N 79.96611°W / 32.80167; -79.9661132°48′6″N 79°57′58″W / 32.80167°N 79.96611°W / 32.80167; -79.9661178002500"National Register Information System"Historic houses of South Carolina"Lowndes Grove""+32° 48' 6.00", −79° 57' 58.00""Lowndes Grove, Charleston County (260 St. Margaret St., Charleston)""Lowndes Grove"The Charleston ExpositionIt Happened in South Carolina"Lowndes Grove (House), Saint Margaret Street & Sixth Avenue, Charleston, Charleston County, SC(Photographs)"Plantations of the Carolina Low Countrye

random experiment with two different functions on unit interval Announcing the arrival of Valued Associate #679: Cesar Manara Planned maintenance scheduled April 23, 2019 at 00:00UTC (8:00pm US/Eastern)Random variable and probability space notionsRandom Walk with EdgesFinding functions where the increase over a random interval is Poisson distributedNumber of days until dayCan an observed event in fact be of zero probability?Unit random processmodels of coins and uniform distributionHow to get the number of successes given $n$ trials , probability $P$ and a random variable $X$Absorbing Markov chain in a computer. Is “almost every” turned into always convergence in computer executions?Stopped random walk is not uniformly integrable

How should I support this large drywall patch? Planned maintenance scheduled April 23, 2019 at 00:00UTC (8:00pm US/Eastern) Announcing the arrival of Valued Associate #679: Cesar Manara Unicorn Meta Zoo #1: Why another podcast?How do I cover large gaps in drywall?How do I keep drywall around a patch from crumbling?Can I glue a second layer of drywall?How to patch long strip on drywall?Large drywall patch: how to avoid bulging seams?Drywall Mesh Patch vs. Bulge? To remove or not to remove?How to fix this drywall job?Prep drywall before backsplashWhat's the best way to fix this horrible drywall patch job?Drywall patching using 3M Patch Plus Primer